Does the integral $int_{0}^{infty} frac{x^3,cos{(x^2-x)}}{1+x^2}$ diverge











up vote
0
down vote

favorite












Does the integral
$$J:=int_{0}^{infty} frac{x^3,cos{(x^2-x)}}{1+x^2}dx $$
diverge ?



If we integrate by parts we find



$$J=lim_{arightarrow +infty} frac{a^3}{(1+a^2)(2a-1)}cos{(a^2-a)} -\
int_{0}^{infty}sin{(x^2-x)}
left( frac{3x^2}{(1+x^2)(2x-1)}-\
frac{2x^4}{(1+x^2)^2(2x-1)}-frac{2x^3}{(1+x^2)(2x-1)^2}right)dx$$



If we integrate by parts again, we find that the latter integral reduces to
a sum of absolutely convergent integrals plus
the boundary term



$$ -lim_{arightarrow +infty},
cos{(a^2-a)}left(frac{3a^2}{(1+a^2)(2a-1)^2}
-\
frac{2a^4}{(1+a^2)^2(2a-1)^2}-frac{2a^3}{(1+a^2)(2a-1)^3}
right)=0.$$



So the question simplifies to the existence of the limit
$lim_{arightarrow +infty} frac{a^3}{(1+a^2)(2a-1)}cos{(a^2-a)}$ which I believe does not exist.



Correct ?










share|cite|improve this question




















  • 1




    Correct. Assuming your math is correct, you can look at your limit and see that for large $a$ the rational bit essentially goes to order 1, and that leaves the cosine which has an undefined limit at infinty.
    – InertialObserver
    Nov 12 at 5:11












  • I am quite sure about the rest. Are you sure the limit in question does not exit ?
    – Medo
    Nov 12 at 5:13










  • This isn't a proof this is an intuition for a proof. But just consider any rational function times a cosine. Unless the rational part goes to zero, the sinusoidal part will always be there, which has a non existent limit yielding a nonexistence. In any case the limit is just going to keep giving you faster and faster cosines unless the polynomial pushes it to zero. You could prob to some perverse L'Hopital magic if you really wanted to.
    – InertialObserver
    Nov 12 at 5:17












  • Dear InertialObserver, thanks a lot for your comments. I know this pretty well. But let me warn you about something: Assume $lim_{xrightarrow x_{0}} f(x)$ exists but $lim_{xrightarrow x_{0}} g(x)$ does not. This does not imply $lim_{xrightarrow x_{0}} f(x) g(x)$ does not exist. ($x_{0}$ can be $infty$). Think of the functions $x$ and $1/x$ and the limits at $infty$. I already know your intuition, and that is why I said "I believe the limit does not exist" in my question. I am looking for a rigorous proof.
    – Medo
    Nov 12 at 5:24










  • @Medo the example you gave in the last comment is not correct. $lim_{xtoinfty}x=infty$ and $lim_{xtoinfty}1/x=0$. Both limits exist.
    – YiFan
    Nov 12 at 5:30

















up vote
0
down vote

favorite












Does the integral
$$J:=int_{0}^{infty} frac{x^3,cos{(x^2-x)}}{1+x^2}dx $$
diverge ?



If we integrate by parts we find



$$J=lim_{arightarrow +infty} frac{a^3}{(1+a^2)(2a-1)}cos{(a^2-a)} -\
int_{0}^{infty}sin{(x^2-x)}
left( frac{3x^2}{(1+x^2)(2x-1)}-\
frac{2x^4}{(1+x^2)^2(2x-1)}-frac{2x^3}{(1+x^2)(2x-1)^2}right)dx$$



If we integrate by parts again, we find that the latter integral reduces to
a sum of absolutely convergent integrals plus
the boundary term



$$ -lim_{arightarrow +infty},
cos{(a^2-a)}left(frac{3a^2}{(1+a^2)(2a-1)^2}
-\
frac{2a^4}{(1+a^2)^2(2a-1)^2}-frac{2a^3}{(1+a^2)(2a-1)^3}
right)=0.$$



So the question simplifies to the existence of the limit
$lim_{arightarrow +infty} frac{a^3}{(1+a^2)(2a-1)}cos{(a^2-a)}$ which I believe does not exist.



Correct ?










share|cite|improve this question




















  • 1




    Correct. Assuming your math is correct, you can look at your limit and see that for large $a$ the rational bit essentially goes to order 1, and that leaves the cosine which has an undefined limit at infinty.
    – InertialObserver
    Nov 12 at 5:11












  • I am quite sure about the rest. Are you sure the limit in question does not exit ?
    – Medo
    Nov 12 at 5:13










  • This isn't a proof this is an intuition for a proof. But just consider any rational function times a cosine. Unless the rational part goes to zero, the sinusoidal part will always be there, which has a non existent limit yielding a nonexistence. In any case the limit is just going to keep giving you faster and faster cosines unless the polynomial pushes it to zero. You could prob to some perverse L'Hopital magic if you really wanted to.
    – InertialObserver
    Nov 12 at 5:17












  • Dear InertialObserver, thanks a lot for your comments. I know this pretty well. But let me warn you about something: Assume $lim_{xrightarrow x_{0}} f(x)$ exists but $lim_{xrightarrow x_{0}} g(x)$ does not. This does not imply $lim_{xrightarrow x_{0}} f(x) g(x)$ does not exist. ($x_{0}$ can be $infty$). Think of the functions $x$ and $1/x$ and the limits at $infty$. I already know your intuition, and that is why I said "I believe the limit does not exist" in my question. I am looking for a rigorous proof.
    – Medo
    Nov 12 at 5:24










  • @Medo the example you gave in the last comment is not correct. $lim_{xtoinfty}x=infty$ and $lim_{xtoinfty}1/x=0$. Both limits exist.
    – YiFan
    Nov 12 at 5:30















up vote
0
down vote

favorite









up vote
0
down vote

favorite











Does the integral
$$J:=int_{0}^{infty} frac{x^3,cos{(x^2-x)}}{1+x^2}dx $$
diverge ?



If we integrate by parts we find



$$J=lim_{arightarrow +infty} frac{a^3}{(1+a^2)(2a-1)}cos{(a^2-a)} -\
int_{0}^{infty}sin{(x^2-x)}
left( frac{3x^2}{(1+x^2)(2x-1)}-\
frac{2x^4}{(1+x^2)^2(2x-1)}-frac{2x^3}{(1+x^2)(2x-1)^2}right)dx$$



If we integrate by parts again, we find that the latter integral reduces to
a sum of absolutely convergent integrals plus
the boundary term



$$ -lim_{arightarrow +infty},
cos{(a^2-a)}left(frac{3a^2}{(1+a^2)(2a-1)^2}
-\
frac{2a^4}{(1+a^2)^2(2a-1)^2}-frac{2a^3}{(1+a^2)(2a-1)^3}
right)=0.$$



So the question simplifies to the existence of the limit
$lim_{arightarrow +infty} frac{a^3}{(1+a^2)(2a-1)}cos{(a^2-a)}$ which I believe does not exist.



Correct ?










share|cite|improve this question















Does the integral
$$J:=int_{0}^{infty} frac{x^3,cos{(x^2-x)}}{1+x^2}dx $$
diverge ?



If we integrate by parts we find



$$J=lim_{arightarrow +infty} frac{a^3}{(1+a^2)(2a-1)}cos{(a^2-a)} -\
int_{0}^{infty}sin{(x^2-x)}
left( frac{3x^2}{(1+x^2)(2x-1)}-\
frac{2x^4}{(1+x^2)^2(2x-1)}-frac{2x^3}{(1+x^2)(2x-1)^2}right)dx$$



If we integrate by parts again, we find that the latter integral reduces to
a sum of absolutely convergent integrals plus
the boundary term



$$ -lim_{arightarrow +infty},
cos{(a^2-a)}left(frac{3a^2}{(1+a^2)(2a-1)^2}
-\
frac{2a^4}{(1+a^2)^2(2a-1)^2}-frac{2a^3}{(1+a^2)(2a-1)^3}
right)=0.$$



So the question simplifies to the existence of the limit
$lim_{arightarrow +infty} frac{a^3}{(1+a^2)(2a-1)}cos{(a^2-a)}$ which I believe does not exist.



Correct ?







real-analysis analysis proof-verification improper-integrals contour-integration






share|cite|improve this question















share|cite|improve this question













share|cite|improve this question




share|cite|improve this question








edited Nov 12 at 5:09

























asked Nov 12 at 5:07









Medo

612213




612213








  • 1




    Correct. Assuming your math is correct, you can look at your limit and see that for large $a$ the rational bit essentially goes to order 1, and that leaves the cosine which has an undefined limit at infinty.
    – InertialObserver
    Nov 12 at 5:11












  • I am quite sure about the rest. Are you sure the limit in question does not exit ?
    – Medo
    Nov 12 at 5:13










  • This isn't a proof this is an intuition for a proof. But just consider any rational function times a cosine. Unless the rational part goes to zero, the sinusoidal part will always be there, which has a non existent limit yielding a nonexistence. In any case the limit is just going to keep giving you faster and faster cosines unless the polynomial pushes it to zero. You could prob to some perverse L'Hopital magic if you really wanted to.
    – InertialObserver
    Nov 12 at 5:17












  • Dear InertialObserver, thanks a lot for your comments. I know this pretty well. But let me warn you about something: Assume $lim_{xrightarrow x_{0}} f(x)$ exists but $lim_{xrightarrow x_{0}} g(x)$ does not. This does not imply $lim_{xrightarrow x_{0}} f(x) g(x)$ does not exist. ($x_{0}$ can be $infty$). Think of the functions $x$ and $1/x$ and the limits at $infty$. I already know your intuition, and that is why I said "I believe the limit does not exist" in my question. I am looking for a rigorous proof.
    – Medo
    Nov 12 at 5:24










  • @Medo the example you gave in the last comment is not correct. $lim_{xtoinfty}x=infty$ and $lim_{xtoinfty}1/x=0$. Both limits exist.
    – YiFan
    Nov 12 at 5:30
















  • 1




    Correct. Assuming your math is correct, you can look at your limit and see that for large $a$ the rational bit essentially goes to order 1, and that leaves the cosine which has an undefined limit at infinty.
    – InertialObserver
    Nov 12 at 5:11












  • I am quite sure about the rest. Are you sure the limit in question does not exit ?
    – Medo
    Nov 12 at 5:13










  • This isn't a proof this is an intuition for a proof. But just consider any rational function times a cosine. Unless the rational part goes to zero, the sinusoidal part will always be there, which has a non existent limit yielding a nonexistence. In any case the limit is just going to keep giving you faster and faster cosines unless the polynomial pushes it to zero. You could prob to some perverse L'Hopital magic if you really wanted to.
    – InertialObserver
    Nov 12 at 5:17












  • Dear InertialObserver, thanks a lot for your comments. I know this pretty well. But let me warn you about something: Assume $lim_{xrightarrow x_{0}} f(x)$ exists but $lim_{xrightarrow x_{0}} g(x)$ does not. This does not imply $lim_{xrightarrow x_{0}} f(x) g(x)$ does not exist. ($x_{0}$ can be $infty$). Think of the functions $x$ and $1/x$ and the limits at $infty$. I already know your intuition, and that is why I said "I believe the limit does not exist" in my question. I am looking for a rigorous proof.
    – Medo
    Nov 12 at 5:24










  • @Medo the example you gave in the last comment is not correct. $lim_{xtoinfty}x=infty$ and $lim_{xtoinfty}1/x=0$. Both limits exist.
    – YiFan
    Nov 12 at 5:30










1




1




Correct. Assuming your math is correct, you can look at your limit and see that for large $a$ the rational bit essentially goes to order 1, and that leaves the cosine which has an undefined limit at infinty.
– InertialObserver
Nov 12 at 5:11






Correct. Assuming your math is correct, you can look at your limit and see that for large $a$ the rational bit essentially goes to order 1, and that leaves the cosine which has an undefined limit at infinty.
– InertialObserver
Nov 12 at 5:11














I am quite sure about the rest. Are you sure the limit in question does not exit ?
– Medo
Nov 12 at 5:13




I am quite sure about the rest. Are you sure the limit in question does not exit ?
– Medo
Nov 12 at 5:13












This isn't a proof this is an intuition for a proof. But just consider any rational function times a cosine. Unless the rational part goes to zero, the sinusoidal part will always be there, which has a non existent limit yielding a nonexistence. In any case the limit is just going to keep giving you faster and faster cosines unless the polynomial pushes it to zero. You could prob to some perverse L'Hopital magic if you really wanted to.
– InertialObserver
Nov 12 at 5:17






This isn't a proof this is an intuition for a proof. But just consider any rational function times a cosine. Unless the rational part goes to zero, the sinusoidal part will always be there, which has a non existent limit yielding a nonexistence. In any case the limit is just going to keep giving you faster and faster cosines unless the polynomial pushes it to zero. You could prob to some perverse L'Hopital magic if you really wanted to.
– InertialObserver
Nov 12 at 5:17














Dear InertialObserver, thanks a lot for your comments. I know this pretty well. But let me warn you about something: Assume $lim_{xrightarrow x_{0}} f(x)$ exists but $lim_{xrightarrow x_{0}} g(x)$ does not. This does not imply $lim_{xrightarrow x_{0}} f(x) g(x)$ does not exist. ($x_{0}$ can be $infty$). Think of the functions $x$ and $1/x$ and the limits at $infty$. I already know your intuition, and that is why I said "I believe the limit does not exist" in my question. I am looking for a rigorous proof.
– Medo
Nov 12 at 5:24




Dear InertialObserver, thanks a lot for your comments. I know this pretty well. But let me warn you about something: Assume $lim_{xrightarrow x_{0}} f(x)$ exists but $lim_{xrightarrow x_{0}} g(x)$ does not. This does not imply $lim_{xrightarrow x_{0}} f(x) g(x)$ does not exist. ($x_{0}$ can be $infty$). Think of the functions $x$ and $1/x$ and the limits at $infty$. I already know your intuition, and that is why I said "I believe the limit does not exist" in my question. I am looking for a rigorous proof.
– Medo
Nov 12 at 5:24












@Medo the example you gave in the last comment is not correct. $lim_{xtoinfty}x=infty$ and $lim_{xtoinfty}1/x=0$. Both limits exist.
– YiFan
Nov 12 at 5:30






@Medo the example you gave in the last comment is not correct. $lim_{xtoinfty}x=infty$ and $lim_{xtoinfty}1/x=0$. Both limits exist.
– YiFan
Nov 12 at 5:30












2 Answers
2






active

oldest

votes

















up vote
0
down vote













The integral is divergent because the limit does not exist.
Let $f(a)=frac{a^3,cos{(a^2-a)}}{(1+a^2)(2a-1)}$.



Take the sequence $a_{n}=:frac{1+sqrt{1+8pi n}}{2}$
that diverges to infinity and satisfies $a_{n}^2-a_{n}=2npi$.
Then $f(a_{n})rightarrow frac{1}{2}$.



Now, take the sequence $b_{n}:=frac{1+sqrt{1+4(2n+1)pi }}{2}$ that also diverges to infinity and satisfies $b_{n}^2-b_{n}=(2n+1)pi$. Then
$f(b_{n})rightarrow -frac{1}{2}$.






share|cite|improve this answer






























    up vote
    0
    down vote













    Also you can do that: an improper integral $lim_{rtoinfty}int_0^r f(x), dx$ converges if and only if for each $epsilon>0$ there is some $M>0$ such that $left|int_a^b f(x), dxright|<epsilon$ for all pairs $a,bge M$.



    Now note that $cos(x)gesqrt 2/2$ when $xin(-pi/4+2kpi,pi/4+2kpi)$ for any $kinBbb Z$. Now observe that



    $$x^2-x=alpha,land, x,alpha>0iff x=frac{1+sqrt{1+4alpha}}2$$



    Now for $alpha=2pi npmpi/4$ we set $a_{npm}:=frac12(1+sqrt{1+pi(4npm 1)})$ and in your case you have that



    $$begin{align}I_n&:=left|int_{a_{n-}}^{a_{n+}}f(x), dxright|gefrac{sqrt2}2int_{a_{n-}}^{a_{n+}}frac{x^3}{1+x^2}, dx\
    &gefrac{sqrt 2}2cdot(a_{n+}-a_{n-})min_{xin[a_{n-},a_{n+}]}frac{x}2,&text{because }frac{x^3}{1+x^2}ge frac{x}2text{ when }xge 1\
    &=frac{sqrt 2}2(a_{n+}-a_{n-})cdotfrac{a_{n+}+a_{n-}}{a_{n+}+a_{n-}}cdotfrac{a_{n-}}2\
    &gefrac{sqrt 2}2cdotfrac{a^2_{n+}-a^2_{n-}}{2a_{n+}}cdotfrac{a_{n-}}2,&text{ because } 2a_{n+}ge a_{n-}+a_{n+}\
    &gefrac{pisqrt 2}{16}cdotsqrt{frac{1+pi(4n-1)}{1+pi(4n+1)}}end{align}$$



    Hence $lim_{ntoinfty}I_ngepisqrt 2/16$, so we can conclude that the integral doesn't converge.






    share|cite|improve this answer























    • So your proof is correct if we want to show that the intergral in question does not converge to zero. In this case I would recommend adding a last sentence to clarify the logic of your proof: You have proved that there exists $c=frac{sqrt 2}{32} pi$ ( notice that $sqrt{frac{1+pi(4n-1)}{1+pi(4n+1)}}>frac{1}{2}$ for all $ngeq 1$) such that for every $M>0 $ there exist s a large enough $n$ (equivalently there exist $a=a_{n_{-}}$ and $b=a_{n_{+}}$ ) such that $a,b> M$ but $left| int_{a}^{b}f(x)dxright|>c$.
      – Medo
      Nov 12 at 18:08










    • Im showing that the sequence of differences $$left|int_0^{a_{n+}}f(x),dx-int_0^{a_{n-}}f(x), dxright|to C>0$$ as $ntoinfty$, hence the sequence defined by $int_0^{a_{n-}},int_0^{a_{n+}},int_0^{a_{n+1-}},...$ is not Cauchy, and because this sequence converges to $int_0^infty$ then the integral diverges
      – Masacroso
      Nov 12 at 18:13












    • no, re-read the answer please. I just need to show that $I_nto C>0$. There is no limit involved here because Im using a Cauchy sequence. Take a look here for a longer explanation
      – Masacroso
      Nov 12 at 18:23












    • You lost me. $I_{n}rightarrow c$ means $I_{n}-I_{n+1}rightarrow 0$ means $I_{n}-I_{m}rightarrow 0$, i.e., $I_{n}$ is Cauchy and by completeness of $mathbb{R}$ convergent!
      – Medo
      Nov 12 at 18:26










    • Okay. The criteria you provided for convergence checks out. Your answer proves the divergence of the integral. Thanks a lot. If you look at my first comment, you see we do not need to use any sequential criterion. The definition you gave is enough.
      – Medo
      Nov 12 at 18:35













    Your Answer





    StackExchange.ifUsing("editor", function () {
    return StackExchange.using("mathjaxEditing", function () {
    StackExchange.MarkdownEditor.creationCallbacks.add(function (editor, postfix) {
    StackExchange.mathjaxEditing.prepareWmdForMathJax(editor, postfix, [["$", "$"], ["\\(","\\)"]]);
    });
    });
    }, "mathjax-editing");

    StackExchange.ready(function() {
    var channelOptions = {
    tags: "".split(" "),
    id: "69"
    };
    initTagRenderer("".split(" "), "".split(" "), channelOptions);

    StackExchange.using("externalEditor", function() {
    // Have to fire editor after snippets, if snippets enabled
    if (StackExchange.settings.snippets.snippetsEnabled) {
    StackExchange.using("snippets", function() {
    createEditor();
    });
    }
    else {
    createEditor();
    }
    });

    function createEditor() {
    StackExchange.prepareEditor({
    heartbeatType: 'answer',
    convertImagesToLinks: true,
    noModals: true,
    showLowRepImageUploadWarning: true,
    reputationToPostImages: 10,
    bindNavPrevention: true,
    postfix: "",
    imageUploader: {
    brandingHtml: "Powered by u003ca class="icon-imgur-white" href="https://imgur.com/"u003eu003c/au003e",
    contentPolicyHtml: "User contributions licensed under u003ca href="https://creativecommons.org/licenses/by-sa/3.0/"u003ecc by-sa 3.0 with attribution requiredu003c/au003e u003ca href="https://stackoverflow.com/legal/content-policy"u003e(content policy)u003c/au003e",
    allowUrls: true
    },
    noCode: true, onDemand: true,
    discardSelector: ".discard-answer"
    ,immediatelyShowMarkdownHelp:true
    });


    }
    });














     

    draft saved


    draft discarded


















    StackExchange.ready(
    function () {
    StackExchange.openid.initPostLogin('.new-post-login', 'https%3a%2f%2fmath.stackexchange.com%2fquestions%2f2994884%2fdoes-the-integral-int-0-infty-fracx3-cosx2-x1x2-diverge%23new-answer', 'question_page');
    }
    );

    Post as a guest















    Required, but never shown

























    2 Answers
    2






    active

    oldest

    votes








    2 Answers
    2






    active

    oldest

    votes









    active

    oldest

    votes






    active

    oldest

    votes








    up vote
    0
    down vote













    The integral is divergent because the limit does not exist.
    Let $f(a)=frac{a^3,cos{(a^2-a)}}{(1+a^2)(2a-1)}$.



    Take the sequence $a_{n}=:frac{1+sqrt{1+8pi n}}{2}$
    that diverges to infinity and satisfies $a_{n}^2-a_{n}=2npi$.
    Then $f(a_{n})rightarrow frac{1}{2}$.



    Now, take the sequence $b_{n}:=frac{1+sqrt{1+4(2n+1)pi }}{2}$ that also diverges to infinity and satisfies $b_{n}^2-b_{n}=(2n+1)pi$. Then
    $f(b_{n})rightarrow -frac{1}{2}$.






    share|cite|improve this answer



























      up vote
      0
      down vote













      The integral is divergent because the limit does not exist.
      Let $f(a)=frac{a^3,cos{(a^2-a)}}{(1+a^2)(2a-1)}$.



      Take the sequence $a_{n}=:frac{1+sqrt{1+8pi n}}{2}$
      that diverges to infinity and satisfies $a_{n}^2-a_{n}=2npi$.
      Then $f(a_{n})rightarrow frac{1}{2}$.



      Now, take the sequence $b_{n}:=frac{1+sqrt{1+4(2n+1)pi }}{2}$ that also diverges to infinity and satisfies $b_{n}^2-b_{n}=(2n+1)pi$. Then
      $f(b_{n})rightarrow -frac{1}{2}$.






      share|cite|improve this answer

























        up vote
        0
        down vote










        up vote
        0
        down vote









        The integral is divergent because the limit does not exist.
        Let $f(a)=frac{a^3,cos{(a^2-a)}}{(1+a^2)(2a-1)}$.



        Take the sequence $a_{n}=:frac{1+sqrt{1+8pi n}}{2}$
        that diverges to infinity and satisfies $a_{n}^2-a_{n}=2npi$.
        Then $f(a_{n})rightarrow frac{1}{2}$.



        Now, take the sequence $b_{n}:=frac{1+sqrt{1+4(2n+1)pi }}{2}$ that also diverges to infinity and satisfies $b_{n}^2-b_{n}=(2n+1)pi$. Then
        $f(b_{n})rightarrow -frac{1}{2}$.






        share|cite|improve this answer














        The integral is divergent because the limit does not exist.
        Let $f(a)=frac{a^3,cos{(a^2-a)}}{(1+a^2)(2a-1)}$.



        Take the sequence $a_{n}=:frac{1+sqrt{1+8pi n}}{2}$
        that diverges to infinity and satisfies $a_{n}^2-a_{n}=2npi$.
        Then $f(a_{n})rightarrow frac{1}{2}$.



        Now, take the sequence $b_{n}:=frac{1+sqrt{1+4(2n+1)pi }}{2}$ that also diverges to infinity and satisfies $b_{n}^2-b_{n}=(2n+1)pi$. Then
        $f(b_{n})rightarrow -frac{1}{2}$.







        share|cite|improve this answer














        share|cite|improve this answer



        share|cite|improve this answer








        edited Nov 12 at 17:34

























        answered Nov 12 at 6:01









        Medo

        612213




        612213






















            up vote
            0
            down vote













            Also you can do that: an improper integral $lim_{rtoinfty}int_0^r f(x), dx$ converges if and only if for each $epsilon>0$ there is some $M>0$ such that $left|int_a^b f(x), dxright|<epsilon$ for all pairs $a,bge M$.



            Now note that $cos(x)gesqrt 2/2$ when $xin(-pi/4+2kpi,pi/4+2kpi)$ for any $kinBbb Z$. Now observe that



            $$x^2-x=alpha,land, x,alpha>0iff x=frac{1+sqrt{1+4alpha}}2$$



            Now for $alpha=2pi npmpi/4$ we set $a_{npm}:=frac12(1+sqrt{1+pi(4npm 1)})$ and in your case you have that



            $$begin{align}I_n&:=left|int_{a_{n-}}^{a_{n+}}f(x), dxright|gefrac{sqrt2}2int_{a_{n-}}^{a_{n+}}frac{x^3}{1+x^2}, dx\
            &gefrac{sqrt 2}2cdot(a_{n+}-a_{n-})min_{xin[a_{n-},a_{n+}]}frac{x}2,&text{because }frac{x^3}{1+x^2}ge frac{x}2text{ when }xge 1\
            &=frac{sqrt 2}2(a_{n+}-a_{n-})cdotfrac{a_{n+}+a_{n-}}{a_{n+}+a_{n-}}cdotfrac{a_{n-}}2\
            &gefrac{sqrt 2}2cdotfrac{a^2_{n+}-a^2_{n-}}{2a_{n+}}cdotfrac{a_{n-}}2,&text{ because } 2a_{n+}ge a_{n-}+a_{n+}\
            &gefrac{pisqrt 2}{16}cdotsqrt{frac{1+pi(4n-1)}{1+pi(4n+1)}}end{align}$$



            Hence $lim_{ntoinfty}I_ngepisqrt 2/16$, so we can conclude that the integral doesn't converge.






            share|cite|improve this answer























            • So your proof is correct if we want to show that the intergral in question does not converge to zero. In this case I would recommend adding a last sentence to clarify the logic of your proof: You have proved that there exists $c=frac{sqrt 2}{32} pi$ ( notice that $sqrt{frac{1+pi(4n-1)}{1+pi(4n+1)}}>frac{1}{2}$ for all $ngeq 1$) such that for every $M>0 $ there exist s a large enough $n$ (equivalently there exist $a=a_{n_{-}}$ and $b=a_{n_{+}}$ ) such that $a,b> M$ but $left| int_{a}^{b}f(x)dxright|>c$.
              – Medo
              Nov 12 at 18:08










            • Im showing that the sequence of differences $$left|int_0^{a_{n+}}f(x),dx-int_0^{a_{n-}}f(x), dxright|to C>0$$ as $ntoinfty$, hence the sequence defined by $int_0^{a_{n-}},int_0^{a_{n+}},int_0^{a_{n+1-}},...$ is not Cauchy, and because this sequence converges to $int_0^infty$ then the integral diverges
              – Masacroso
              Nov 12 at 18:13












            • no, re-read the answer please. I just need to show that $I_nto C>0$. There is no limit involved here because Im using a Cauchy sequence. Take a look here for a longer explanation
              – Masacroso
              Nov 12 at 18:23












            • You lost me. $I_{n}rightarrow c$ means $I_{n}-I_{n+1}rightarrow 0$ means $I_{n}-I_{m}rightarrow 0$, i.e., $I_{n}$ is Cauchy and by completeness of $mathbb{R}$ convergent!
              – Medo
              Nov 12 at 18:26










            • Okay. The criteria you provided for convergence checks out. Your answer proves the divergence of the integral. Thanks a lot. If you look at my first comment, you see we do not need to use any sequential criterion. The definition you gave is enough.
              – Medo
              Nov 12 at 18:35

















            up vote
            0
            down vote













            Also you can do that: an improper integral $lim_{rtoinfty}int_0^r f(x), dx$ converges if and only if for each $epsilon>0$ there is some $M>0$ such that $left|int_a^b f(x), dxright|<epsilon$ for all pairs $a,bge M$.



            Now note that $cos(x)gesqrt 2/2$ when $xin(-pi/4+2kpi,pi/4+2kpi)$ for any $kinBbb Z$. Now observe that



            $$x^2-x=alpha,land, x,alpha>0iff x=frac{1+sqrt{1+4alpha}}2$$



            Now for $alpha=2pi npmpi/4$ we set $a_{npm}:=frac12(1+sqrt{1+pi(4npm 1)})$ and in your case you have that



            $$begin{align}I_n&:=left|int_{a_{n-}}^{a_{n+}}f(x), dxright|gefrac{sqrt2}2int_{a_{n-}}^{a_{n+}}frac{x^3}{1+x^2}, dx\
            &gefrac{sqrt 2}2cdot(a_{n+}-a_{n-})min_{xin[a_{n-},a_{n+}]}frac{x}2,&text{because }frac{x^3}{1+x^2}ge frac{x}2text{ when }xge 1\
            &=frac{sqrt 2}2(a_{n+}-a_{n-})cdotfrac{a_{n+}+a_{n-}}{a_{n+}+a_{n-}}cdotfrac{a_{n-}}2\
            &gefrac{sqrt 2}2cdotfrac{a^2_{n+}-a^2_{n-}}{2a_{n+}}cdotfrac{a_{n-}}2,&text{ because } 2a_{n+}ge a_{n-}+a_{n+}\
            &gefrac{pisqrt 2}{16}cdotsqrt{frac{1+pi(4n-1)}{1+pi(4n+1)}}end{align}$$



            Hence $lim_{ntoinfty}I_ngepisqrt 2/16$, so we can conclude that the integral doesn't converge.






            share|cite|improve this answer























            • So your proof is correct if we want to show that the intergral in question does not converge to zero. In this case I would recommend adding a last sentence to clarify the logic of your proof: You have proved that there exists $c=frac{sqrt 2}{32} pi$ ( notice that $sqrt{frac{1+pi(4n-1)}{1+pi(4n+1)}}>frac{1}{2}$ for all $ngeq 1$) such that for every $M>0 $ there exist s a large enough $n$ (equivalently there exist $a=a_{n_{-}}$ and $b=a_{n_{+}}$ ) such that $a,b> M$ but $left| int_{a}^{b}f(x)dxright|>c$.
              – Medo
              Nov 12 at 18:08










            • Im showing that the sequence of differences $$left|int_0^{a_{n+}}f(x),dx-int_0^{a_{n-}}f(x), dxright|to C>0$$ as $ntoinfty$, hence the sequence defined by $int_0^{a_{n-}},int_0^{a_{n+}},int_0^{a_{n+1-}},...$ is not Cauchy, and because this sequence converges to $int_0^infty$ then the integral diverges
              – Masacroso
              Nov 12 at 18:13












            • no, re-read the answer please. I just need to show that $I_nto C>0$. There is no limit involved here because Im using a Cauchy sequence. Take a look here for a longer explanation
              – Masacroso
              Nov 12 at 18:23












            • You lost me. $I_{n}rightarrow c$ means $I_{n}-I_{n+1}rightarrow 0$ means $I_{n}-I_{m}rightarrow 0$, i.e., $I_{n}$ is Cauchy and by completeness of $mathbb{R}$ convergent!
              – Medo
              Nov 12 at 18:26










            • Okay. The criteria you provided for convergence checks out. Your answer proves the divergence of the integral. Thanks a lot. If you look at my first comment, you see we do not need to use any sequential criterion. The definition you gave is enough.
              – Medo
              Nov 12 at 18:35















            up vote
            0
            down vote










            up vote
            0
            down vote









            Also you can do that: an improper integral $lim_{rtoinfty}int_0^r f(x), dx$ converges if and only if for each $epsilon>0$ there is some $M>0$ such that $left|int_a^b f(x), dxright|<epsilon$ for all pairs $a,bge M$.



            Now note that $cos(x)gesqrt 2/2$ when $xin(-pi/4+2kpi,pi/4+2kpi)$ for any $kinBbb Z$. Now observe that



            $$x^2-x=alpha,land, x,alpha>0iff x=frac{1+sqrt{1+4alpha}}2$$



            Now for $alpha=2pi npmpi/4$ we set $a_{npm}:=frac12(1+sqrt{1+pi(4npm 1)})$ and in your case you have that



            $$begin{align}I_n&:=left|int_{a_{n-}}^{a_{n+}}f(x), dxright|gefrac{sqrt2}2int_{a_{n-}}^{a_{n+}}frac{x^3}{1+x^2}, dx\
            &gefrac{sqrt 2}2cdot(a_{n+}-a_{n-})min_{xin[a_{n-},a_{n+}]}frac{x}2,&text{because }frac{x^3}{1+x^2}ge frac{x}2text{ when }xge 1\
            &=frac{sqrt 2}2(a_{n+}-a_{n-})cdotfrac{a_{n+}+a_{n-}}{a_{n+}+a_{n-}}cdotfrac{a_{n-}}2\
            &gefrac{sqrt 2}2cdotfrac{a^2_{n+}-a^2_{n-}}{2a_{n+}}cdotfrac{a_{n-}}2,&text{ because } 2a_{n+}ge a_{n-}+a_{n+}\
            &gefrac{pisqrt 2}{16}cdotsqrt{frac{1+pi(4n-1)}{1+pi(4n+1)}}end{align}$$



            Hence $lim_{ntoinfty}I_ngepisqrt 2/16$, so we can conclude that the integral doesn't converge.






            share|cite|improve this answer














            Also you can do that: an improper integral $lim_{rtoinfty}int_0^r f(x), dx$ converges if and only if for each $epsilon>0$ there is some $M>0$ such that $left|int_a^b f(x), dxright|<epsilon$ for all pairs $a,bge M$.



            Now note that $cos(x)gesqrt 2/2$ when $xin(-pi/4+2kpi,pi/4+2kpi)$ for any $kinBbb Z$. Now observe that



            $$x^2-x=alpha,land, x,alpha>0iff x=frac{1+sqrt{1+4alpha}}2$$



            Now for $alpha=2pi npmpi/4$ we set $a_{npm}:=frac12(1+sqrt{1+pi(4npm 1)})$ and in your case you have that



            $$begin{align}I_n&:=left|int_{a_{n-}}^{a_{n+}}f(x), dxright|gefrac{sqrt2}2int_{a_{n-}}^{a_{n+}}frac{x^3}{1+x^2}, dx\
            &gefrac{sqrt 2}2cdot(a_{n+}-a_{n-})min_{xin[a_{n-},a_{n+}]}frac{x}2,&text{because }frac{x^3}{1+x^2}ge frac{x}2text{ when }xge 1\
            &=frac{sqrt 2}2(a_{n+}-a_{n-})cdotfrac{a_{n+}+a_{n-}}{a_{n+}+a_{n-}}cdotfrac{a_{n-}}2\
            &gefrac{sqrt 2}2cdotfrac{a^2_{n+}-a^2_{n-}}{2a_{n+}}cdotfrac{a_{n-}}2,&text{ because } 2a_{n+}ge a_{n-}+a_{n+}\
            &gefrac{pisqrt 2}{16}cdotsqrt{frac{1+pi(4n-1)}{1+pi(4n+1)}}end{align}$$



            Hence $lim_{ntoinfty}I_ngepisqrt 2/16$, so we can conclude that the integral doesn't converge.







            share|cite|improve this answer














            share|cite|improve this answer



            share|cite|improve this answer








            edited Nov 13 at 3:50

























            answered Nov 12 at 6:21









            Masacroso

            12.2k41746




            12.2k41746












            • So your proof is correct if we want to show that the intergral in question does not converge to zero. In this case I would recommend adding a last sentence to clarify the logic of your proof: You have proved that there exists $c=frac{sqrt 2}{32} pi$ ( notice that $sqrt{frac{1+pi(4n-1)}{1+pi(4n+1)}}>frac{1}{2}$ for all $ngeq 1$) such that for every $M>0 $ there exist s a large enough $n$ (equivalently there exist $a=a_{n_{-}}$ and $b=a_{n_{+}}$ ) such that $a,b> M$ but $left| int_{a}^{b}f(x)dxright|>c$.
              – Medo
              Nov 12 at 18:08










            • Im showing that the sequence of differences $$left|int_0^{a_{n+}}f(x),dx-int_0^{a_{n-}}f(x), dxright|to C>0$$ as $ntoinfty$, hence the sequence defined by $int_0^{a_{n-}},int_0^{a_{n+}},int_0^{a_{n+1-}},...$ is not Cauchy, and because this sequence converges to $int_0^infty$ then the integral diverges
              – Masacroso
              Nov 12 at 18:13












            • no, re-read the answer please. I just need to show that $I_nto C>0$. There is no limit involved here because Im using a Cauchy sequence. Take a look here for a longer explanation
              – Masacroso
              Nov 12 at 18:23












            • You lost me. $I_{n}rightarrow c$ means $I_{n}-I_{n+1}rightarrow 0$ means $I_{n}-I_{m}rightarrow 0$, i.e., $I_{n}$ is Cauchy and by completeness of $mathbb{R}$ convergent!
              – Medo
              Nov 12 at 18:26










            • Okay. The criteria you provided for convergence checks out. Your answer proves the divergence of the integral. Thanks a lot. If you look at my first comment, you see we do not need to use any sequential criterion. The definition you gave is enough.
              – Medo
              Nov 12 at 18:35




















            • So your proof is correct if we want to show that the intergral in question does not converge to zero. In this case I would recommend adding a last sentence to clarify the logic of your proof: You have proved that there exists $c=frac{sqrt 2}{32} pi$ ( notice that $sqrt{frac{1+pi(4n-1)}{1+pi(4n+1)}}>frac{1}{2}$ for all $ngeq 1$) such that for every $M>0 $ there exist s a large enough $n$ (equivalently there exist $a=a_{n_{-}}$ and $b=a_{n_{+}}$ ) such that $a,b> M$ but $left| int_{a}^{b}f(x)dxright|>c$.
              – Medo
              Nov 12 at 18:08










            • Im showing that the sequence of differences $$left|int_0^{a_{n+}}f(x),dx-int_0^{a_{n-}}f(x), dxright|to C>0$$ as $ntoinfty$, hence the sequence defined by $int_0^{a_{n-}},int_0^{a_{n+}},int_0^{a_{n+1-}},...$ is not Cauchy, and because this sequence converges to $int_0^infty$ then the integral diverges
              – Masacroso
              Nov 12 at 18:13












            • no, re-read the answer please. I just need to show that $I_nto C>0$. There is no limit involved here because Im using a Cauchy sequence. Take a look here for a longer explanation
              – Masacroso
              Nov 12 at 18:23












            • You lost me. $I_{n}rightarrow c$ means $I_{n}-I_{n+1}rightarrow 0$ means $I_{n}-I_{m}rightarrow 0$, i.e., $I_{n}$ is Cauchy and by completeness of $mathbb{R}$ convergent!
              – Medo
              Nov 12 at 18:26










            • Okay. The criteria you provided for convergence checks out. Your answer proves the divergence of the integral. Thanks a lot. If you look at my first comment, you see we do not need to use any sequential criterion. The definition you gave is enough.
              – Medo
              Nov 12 at 18:35


















            So your proof is correct if we want to show that the intergral in question does not converge to zero. In this case I would recommend adding a last sentence to clarify the logic of your proof: You have proved that there exists $c=frac{sqrt 2}{32} pi$ ( notice that $sqrt{frac{1+pi(4n-1)}{1+pi(4n+1)}}>frac{1}{2}$ for all $ngeq 1$) such that for every $M>0 $ there exist s a large enough $n$ (equivalently there exist $a=a_{n_{-}}$ and $b=a_{n_{+}}$ ) such that $a,b> M$ but $left| int_{a}^{b}f(x)dxright|>c$.
            – Medo
            Nov 12 at 18:08




            So your proof is correct if we want to show that the intergral in question does not converge to zero. In this case I would recommend adding a last sentence to clarify the logic of your proof: You have proved that there exists $c=frac{sqrt 2}{32} pi$ ( notice that $sqrt{frac{1+pi(4n-1)}{1+pi(4n+1)}}>frac{1}{2}$ for all $ngeq 1$) such that for every $M>0 $ there exist s a large enough $n$ (equivalently there exist $a=a_{n_{-}}$ and $b=a_{n_{+}}$ ) such that $a,b> M$ but $left| int_{a}^{b}f(x)dxright|>c$.
            – Medo
            Nov 12 at 18:08












            Im showing that the sequence of differences $$left|int_0^{a_{n+}}f(x),dx-int_0^{a_{n-}}f(x), dxright|to C>0$$ as $ntoinfty$, hence the sequence defined by $int_0^{a_{n-}},int_0^{a_{n+}},int_0^{a_{n+1-}},...$ is not Cauchy, and because this sequence converges to $int_0^infty$ then the integral diverges
            – Masacroso
            Nov 12 at 18:13






            Im showing that the sequence of differences $$left|int_0^{a_{n+}}f(x),dx-int_0^{a_{n-}}f(x), dxright|to C>0$$ as $ntoinfty$, hence the sequence defined by $int_0^{a_{n-}},int_0^{a_{n+}},int_0^{a_{n+1-}},...$ is not Cauchy, and because this sequence converges to $int_0^infty$ then the integral diverges
            – Masacroso
            Nov 12 at 18:13














            no, re-read the answer please. I just need to show that $I_nto C>0$. There is no limit involved here because Im using a Cauchy sequence. Take a look here for a longer explanation
            – Masacroso
            Nov 12 at 18:23






            no, re-read the answer please. I just need to show that $I_nto C>0$. There is no limit involved here because Im using a Cauchy sequence. Take a look here for a longer explanation
            – Masacroso
            Nov 12 at 18:23














            You lost me. $I_{n}rightarrow c$ means $I_{n}-I_{n+1}rightarrow 0$ means $I_{n}-I_{m}rightarrow 0$, i.e., $I_{n}$ is Cauchy and by completeness of $mathbb{R}$ convergent!
            – Medo
            Nov 12 at 18:26




            You lost me. $I_{n}rightarrow c$ means $I_{n}-I_{n+1}rightarrow 0$ means $I_{n}-I_{m}rightarrow 0$, i.e., $I_{n}$ is Cauchy and by completeness of $mathbb{R}$ convergent!
            – Medo
            Nov 12 at 18:26












            Okay. The criteria you provided for convergence checks out. Your answer proves the divergence of the integral. Thanks a lot. If you look at my first comment, you see we do not need to use any sequential criterion. The definition you gave is enough.
            – Medo
            Nov 12 at 18:35






            Okay. The criteria you provided for convergence checks out. Your answer proves the divergence of the integral. Thanks a lot. If you look at my first comment, you see we do not need to use any sequential criterion. The definition you gave is enough.
            – Medo
            Nov 12 at 18:35




















             

            draft saved


            draft discarded



















































             


            draft saved


            draft discarded














            StackExchange.ready(
            function () {
            StackExchange.openid.initPostLogin('.new-post-login', 'https%3a%2f%2fmath.stackexchange.com%2fquestions%2f2994884%2fdoes-the-integral-int-0-infty-fracx3-cosx2-x1x2-diverge%23new-answer', 'question_page');
            }
            );

            Post as a guest















            Required, but never shown





















































            Required, but never shown














            Required, but never shown












            Required, but never shown







            Required, but never shown

































            Required, but never shown














            Required, but never shown












            Required, but never shown







            Required, but never shown







            Popular posts from this blog

            Biblatex bibliography style without URLs when DOI exists (in Overleaf with Zotero bibliography)

            ComboBox Display Member on multiple fields

            Is it possible to collect Nectar points via Trainline?